RegistrierenRegistrieren   LoginLogin   FAQFAQ    SuchenSuchen   
Seminararbeit - gravitationsfreier Punkt - Seite 2
Gehe zu Seite Zurück  1, 2 
Neue Frage »
Antworten »
    Foren-Übersicht -> Mechanik
Autor Nachricht
gnt
Gast





Beitrag gnt Verfasst am: 23. Jun 2018 13:53    Titel: Antworten mit Zitat

Eine geometrische Überlegung:

1. Gleichgewichtspunkte ergeben sich in Gravitationskraftfeldern durch Neutralisation der Vektorfelder, was nur dann erfolgt, wenn mindestens zwei Felder überlagert werden. Daraus folgt, dass bei n Massenpunkten maximal n(n-1)/2 Gleichgewichtspunkte existieren können.

2. Befinden sich alle Massenpunkte am gleichen Ort, existiert natürlich kein Gleichgewichtspunkt (abgesehen von eben diesem einen Ort - ich weiß nicht, wie die Aufgabe für den Fall gestellt ist). Dieser Fall ist zwar eher unsinnig, weil das einer einzelnen Masse entspricht, führt aber zu einer minimalen Zahl Gleichgewichtspunkte von 0.

3. Platziert man n Massenpunkte mit identischer Masse in Form eines gleichseitigen n-Ecks, existiert offensichtlich stets nur 1 Gleichgewichtspunkt.

4. Aus (3) in Verbindung mit (1) kann man folgern, dass die Symmetrie der Massen m und Massenanordnung, mit Orten x, eine wesentliche Rolle spielt. Je mehr Symmetrien man in m und x findet, desto weniger verschiedene Gleichgewichtspunkte existieren. Evtl. muss man hier nur abzählen, wie viele identische Permutationen (m,x) es für die n Massenpunkte gibt, und das von n(n-1)/2 (oder bei allen Permutationen von n(n-1)) abziehen.
Dann sollte man eigentlich die Zahl der Gleichgewichtspunkte haben.

Ist etwas falsch, oder habe ich etwas übersehen?
TomS
Moderator


Anmeldungsdatum: 20.03.2009
Beiträge: 18051

Beitrag TomS Verfasst am: 23. Jun 2018 14:38    Titel: Antworten mit Zitat

gnt hat Folgendes geschrieben:
1. Gleichgewichtspunkte ergeben sich in Gravitationskraftfeldern durch Neutralisation der Vektorfelder, was nur dann erfolgt, wenn mindestens zwei Felder überlagert werden. Daraus folgt, dass bei n Massenpunkten maximal n(n-1)/2 Gleichgewichtspunkte existieren können.

Das Argument berücksichtigt nicht die Form der Vektorfelder. Man kann leicht Vektorfelder konstruieren, für die Ort und Anzahl der Gleichgewichtspunkte beliebig sind.

_________________
Niels Bohr brainwashed a whole generation of theorists into thinking that the job (interpreting quantum theory) was done 50 years ago.
gnt
Gast





Beitrag gnt Verfasst am: 23. Jun 2018 15:06    Titel: Antworten mit Zitat

TomS hat Folgendes geschrieben:
gnt hat Folgendes geschrieben:
1. Gleichgewichtspunkte ergeben sich in Gravitationskraftfeldern durch Neutralisation der Vektorfelder, was nur dann erfolgt, wenn mindestens zwei Felder überlagert werden. Daraus folgt, dass bei n Massenpunkten maximal n(n-1)/2 Gleichgewichtspunkte existieren können.

Das Argument berücksichtigt nicht die Form der Vektorfelder. Man kann leicht Vektorfelder konstruieren, für die Ort und Anzahl der Gleichgewichtspunkte beliebig sind.

Das verstehe ich leider nicht. Es geht dabei ja nur um die maximale Zahl von Gleichgewichtspunkten bei n Massenpunkten unter der Annahme dass alle m und x beliebig sind.
Für jeden Massenpunkt gilt , mit . Ich sehe nicht, wie man für bestimmte n Massenpunkte daraus beliebig viele Gleichgewichtspunkte konstruieren könnte. - Beliebig viele verschiedene Überlagerungen von gibt es natürlich, aber darum ging es mir nicht.
Es gilt doch offensichtlich:
n=1: 0
n=2: 1
n=3: 3
n=4: 6
VeryApe



Anmeldungsdatum: 10.02.2008
Beiträge: 3247

Beitrag VeryApe Verfasst am: 23. Jun 2018 16:14    Titel: Antworten mit Zitat

index_razor hat Folgendes geschrieben:
Worauf soll deine Rechnung denn jetzt hinauslaufen?


Ich hatte mir es so vorgestellt.

Ich nehme irgendeinen Punkt 0 her, lege eine Gerade fest auf der der Punkt P3 (in meinen Fall P3, welcher Punkt ob P1 oder P2 ist egal) liegen soll.

Ich bestimme aber noch nicht auf welchen Abstand er liegen soll, also den Betrag noch nicht,

dann lege ich da ein Koordinatensystem rein mit Ausrichtung auf diese gerade in y (siehe Skizze) und überlege jetzt wie müssen r1 und r2 vom Punkt 0 weglaufen sodass ich den Punkt 0 als kräftefrei erhalten kann.

Dazu zeichne ich zwei Geraden von Punkt 0 weg, also r1 und r2 deren winkeln phi 1 und phi2 auf das Koordinatensystem und stelle die Kräftegleichung in x auf. Die resultiernde Kraft in x muß mal null sein. Das ist eine Forderung an einen kräftefreien Punkt.

dadurch erhalte ich mal die Gleichung cos phi 2=- .... cosphi
siehe vorher.

gehe ich nun her und lege mal fest das der Winkel phi1 im ersten Quartal liegen soll habe ich einen positiven cos phi wert. lege ich nun die Massen m1 und m2 sowie r2 und r1 fest.
kann ich mir cos phi 2 errechnen. das negativ sein muß und somit im 2 Quartal liegen kann. (Die Frage ist noch was ist wenn cosphi vom Betrag her größer als eins ist.)

Dadurch hab ich schon mal P1 und P2 Punkte konstruiert die Kräfte bezüglich 0 ausspucken die sich in x aufheben.

wei schauts jetzt mit den Kraftanteilen in y aus.
da beide winkeln im 1 Quartal sowie im 2 Quartal liegen ist jeweils der sinus phi positiv was bedeutet die ziehen in positiver y Richtung.

Diese 2 y Kraft Komponenten der Punkte P1 und P2 müssen von der Gravitationskraft des dritten Punktes aufgehoben werden und die wirkt aber vollständig in y Richtung denn so hatte ich ja die Koordinaten gewählt.

und dazu brauche ich nur den Betrag von r3 anpassen und ich hätte dreiecke konsturiert in denen der Punkt O liegt und kräftefrei ist.

zu jeden gewählten r1 und r2 und phi1 positiv(1 Quartal) ergibt sich ein bestimmtes phi2 (negativ 2 Quartal) und r3 und somit lässt sich ein Dreieck konstruieren.

Das war eigentlich mal mein Gedankengang.

_________________
WAS IST LOS IN EUROPA? https://www.youtube.com/watch?v=a9mduhSSC5w
TomS
Moderator


Anmeldungsdatum: 20.03.2009
Beiträge: 18051

Beitrag TomS Verfasst am: 23. Jun 2018 17:57    Titel: Antworten mit Zitat

@gnt: in jedem Punkt der Ebene schneiden sich genau n Feldlinien, ausgehend von n Massen; warum sollen nun genau n(n-1)/2 Punkte ausgezeichnet sein? anschaulich ist das klar, aber wir beweist du das? und selbst wenn das funktioniert, dann reicht ein rein kombinatorisches Argument sicher nicht aus; man kann sicher Felder konstruieren, für die das nicht gilt, aber dein Argument berücksichtigt die Form des Feldes nicht; daher ist es zumindest unvollständig.
_________________
Niels Bohr brainwashed a whole generation of theorists into thinking that the job (interpreting quantum theory) was done 50 years ago.
gnt
Gast





Beitrag gnt Verfasst am: 23. Jun 2018 19:08    Titel: Antworten mit Zitat

TomS hat Folgendes geschrieben:
@gnt: in jedem Punkt der Ebene schneiden sich genau n Feldlinien, ausgehend von n Massen; ...

Ja, aber das ist für meine Betrachtung nicht von Bedeutung.

TomS hat Folgendes geschrieben:
...warum sollen nun genau n(n-1)/2 Punkte ausgezeichnet sein? anschaulich ist das klar, aber wir beweist du das?

Mache ich mir das zu einfach(?):
Schaut man sich das Problem mit zwei Massen A und B an, ist offensichtlich, dass es genau einen Gleichgewichtspunkt G(A,B) gibt, der je nach Größe der Massen näher an A oder B liegt. Die Massen sind bei der Maximalanzahl nicht von Bedeutung; sie können die Maximalzahl nur reduzieren.
Fügt man eine dritte Masse C hinzu, kann durch diese G(A,B) nur verschoben oder eliminiert werden; es werden aus G(A,B) aber niemals zwei Punkte - das ist der Kern.
Die gleiche Betrachtung gilt für G(A,C), wenn man A,C hat, und B hinzu nimmt, und für G(B,C), wenn man B,C hat, und A hinzu nimmt.
In eben dieser Weise kann man eine Masse D, E, F... hinzu fügen.
Ich sehe nicht, was man hier noch mathematisch beweisen müsste - die theoretische Maximalzahl der Gleichgewichtspunkte ist dieer Betrachtung zufolge n(n-1)/2.

TomS hat Folgendes geschrieben:
...man kann sicher Felder konstruieren, für die das nicht gilt, aber dein Argument berücksichtigt die Form des Feldes nicht; daher ist es zumindest unvollständig.

Es geht mir ja bei diesem Teil nur um die Maximalzahl von Gleichgewichtspunkten; die Parameter (Massen und Orte) sind dabei irrelevant.
Oder meinst Du mit "Form des Feldes" Felder, die nicht proportional zu -1/r² sind?
TomS
Moderator


Anmeldungsdatum: 20.03.2009
Beiträge: 18051

Beitrag TomS Verfasst am: 24. Jun 2018 11:23    Titel: Antworten mit Zitat

gnt hat Folgendes geschrieben:
Mache ich mir das zu einfach(?)

Ich glaube schon.

gnt hat Folgendes geschrieben:
Schaut man sich das Problem mit zwei Massen A und B an, ist offensichtlich, dass es genau einen Gleichgewichtspunkt G(A,B) gibt, der je nach Größe der Massen näher an A oder B liegt.

Ja.

gnt hat Folgendes geschrieben:
Fügt man eine dritte Masse C hinzu, kann durch diese G(A,B) nur verschoben oder eliminiert werden; es werden aus G(A,B) aber niemals zwei Punkte - das ist der Kern.

Warum?

gnt hat Folgendes geschrieben:
Ich sehe nicht, was man hier noch mathematisch beweisen müsste - die theoretische Maximalzahl der Gleichgewichtspunkte ist dieer Betrachtung zufolge n(n-1)/2.

Ich sehe nicht, dass du überhaupt etwas bewiesen hast, denn

gnt hat Folgendes geschrieben:
TomS hat Folgendes geschrieben:
...man kann sicher Felder konstruieren, für die das nicht gilt, aber dein Argument berücksichtigt die Form des Feldes nicht; daher ist es zumindest unvollständig.

Oder meinst Du mit "Form des Feldes" Felder, die nicht proportional zu -1/r² sind?

Ja.

Du wirst mir sicher zustimmen, dass wenn ich n beliebige Felder nehme, ich über die Anzahl der Gleichgewichtspunkte nichts sagen kann. Jede beliebige Zahl zwischen Null und überabzählbar unendlich ist möglich.

In deinem Fall sollen es nun genau n(n-1)/2 sein. Das muss also etwas mit der speziellen Form des hier vorliegenden Feldes zu tun haben. Du verwendest in deiner Argumentation jedoch nie diese spezielle Form des Feldes. Damit kann deine Argumentation nicht stimmen, denn so wie du sie formuliert hast, gilt sie für beliebige Felder.

_________________
Niels Bohr brainwashed a whole generation of theorists into thinking that the job (interpreting quantum theory) was done 50 years ago.
TomS
Moderator


Anmeldungsdatum: 20.03.2009
Beiträge: 18051

Beitrag TomS Verfasst am: 24. Jun 2018 12:00    Titel: Antworten mit Zitat

Nach etwas suchen: es handelt sich um das mathematische Gebiet der Bifurkationstheorie

https://en.m.wikipedia.org/wiki/Bifurcation_theory

Bifurcation theory is the mathematical study of changes in the qualitative or topological structure of a given family, such as the integral curves of a family of vector fields, and the solutions of a family of differential equations. Most commonly applied to the mathematical study of dynamical systems, a bifurcation occurs when a small smooth change made to the parameter values (the bifurcation parameters) of a system causes a sudden 'qualitative' or topological change in its behaviour. Bifurcations occur in both continuous systems (described by ODEs, DDEs or PDEs) and discrete systems (described by maps). The name "bifurcation" was first introduced by Henri Poincaré in 1885 in the first paper in mathematics showing such a behavior. Henri Poincaré also later named various types of stationary points and classified them.

In mathematics, an integral curve is a parametric curve that represents a specific solution to an ordinary differential equation ... If the differential equation is represented as a vector field or slope field, then the corresponding integral curves are tangent to the field at each point.

Integral curves are known by various other names, depending on the nature and interpretation of the differential equation or vector field. In physics, integral curves for an electric field or magnetic field are known as field lines ...

_________________
Niels Bohr brainwashed a whole generation of theorists into thinking that the job (interpreting quantum theory) was done 50 years ago.
gnt
Gast





Beitrag gnt Verfasst am: 24. Jun 2018 14:05    Titel: Antworten mit Zitat

Es ist richtig, dass meine Überlegung nicht für jeden Potentialverlauf gilt - ich habe nur über klassische Gravitationsfelder nachgedacht.

TomS hat Folgendes geschrieben:
gnt hat Folgendes geschrieben:
Fügt man eine dritte Masse C hinzu, kann durch diese G(A,B) nur verschoben oder eliminiert werden; es werden aus G(A,B) aber niemals zwei Punkte - das ist der Kern.

Warum?

Betrachtet man das Problem über Potentiale, so sind Gleichgewichtspunkte Sattelpunkte. Die beiden Hauptdiagonalen der Sattelflächen sind entgegengesetzt gekrümmt - eine n-förmig (zwischen zwei Massen) und eine u-förmig.

Begibt man sich an einen Ort, so dass man die u-Form sieht, und fügt dort eine neue Masse hinzu, so führt die radiale Komponente, weil diese monoton steigend ist, lediglich zu einer Verschiebung des Sattelpunktes weg vom Betrachter; die beiden tangentialen Komponenten verstärken die u-Form der Sattelfläche lediglich. Insgesamt kommt es also nur zu einer Verschiebung des Sattelpunktes. Außerdem können neue Sattelpunkte zwischen den vorhandenen und der neuen Masse entstehen, aber das hat in meiner Formel nichts mit den bereits existierenden Sattelpunkten zu tun.

Begibt man sich dagegen an einen Ort, von dem aus man die n-Form sieht, und fügt dort eine neue Masse hinzu, führt das je nach Entfernung und Masse zu a) die radiale Komponente lässt den alten Sattelpunkt wegen der monotonen Steigung verschwinden, und die tangentialen Komponenten bilden neue Sattelpunkte zwischen der neuen und den alten Massen, oder b) der alte Sattelpunkt wird in Richtung Betrachter verschoben, und es entsteht wegen den tangentialen Komponenten ein neuer Sattelpunkt.

Insgesamt kann ich also nur erkennen, dass durch Hinzufügen einer n-ten Masse maximal n-1 neue Sattelpunkte entstehen können, und dass bereits vorhandene verschoben oder eliminiert werden können.
TomS
Moderator


Anmeldungsdatum: 20.03.2009
Beiträge: 18051

Beitrag TomS Verfasst am: 24. Jun 2018 15:44    Titel: Antworten mit Zitat

Anschaulich, aber kein Beweis
_________________
Niels Bohr brainwashed a whole generation of theorists into thinking that the job (interpreting quantum theory) was done 50 years ago.
Myon



Anmeldungsdatum: 04.12.2013
Beiträge: 5863

Beitrag Myon Verfasst am: 24. Jun 2018 15:49    Titel: Antworten mit Zitat

Mich wundert einfach immer noch, dass es so schwierig scheint, Literatur zu diesem Problem zu finden. Auch in der Elektrostatik sollte das Problem doch wichtig sein (Gleichgwichtspunkte bei bestimmten Ladungskonfigurationen o.ä.).
jh8979
Moderator


Anmeldungsdatum: 10.07.2012
Beiträge: 8581

Beitrag jh8979 Verfasst am: 24. Jun 2018 16:20    Titel: Antworten mit Zitat

Myon hat Folgendes geschrieben:
Mich wundert einfach immer noch, dass es so schwierig scheint, Literatur zu diesem Problem zu finden. Auch in der Elektrostatik sollte das Problem doch wichtig sein (Gleichgwichtspunkte bei bestimmten Ladungskonfigurationen o.ä.).

Die Gleichgewichtspunkte sind instabil, weil es immer Sattelpunkte sind (folgt aus dem Maximumprinzip für harmonische Funktionen). Daher ist der praktische Nutzen und das Interesse an diesem Problem wohl eher gering.
TomS
Moderator


Anmeldungsdatum: 20.03.2009
Beiträge: 18051

Beitrag TomS Verfasst am: 24. Jun 2018 16:34    Titel: Antworten mit Zitat

jh8979 hat Folgendes geschrieben:
Die Gleichgewichtspunkte sind instabil, weil es immer Sattelpunkte sind ...

Guter Punkt. Das entspricht Earnshaw's Theorem
https://en.m.wikipedia.org/wiki/Earnshaw%27s_theorem

_________________
Niels Bohr brainwashed a whole generation of theorists into thinking that the job (interpreting quantum theory) was done 50 years ago.
gnt
Gast





Beitrag gnt Verfasst am: 24. Jun 2018 16:53    Titel: Antworten mit Zitat

TomS hat Folgendes geschrieben:
Anschaulich, aber kein Beweis

Vielleicht so: Wenn aus einem Sattelpunkt mehrere werden könnten, müssten, wenn man eine infinitesimale Masse hinzu fügt, mindestens zwei neue, infinitesimal benachbarte Sattelpunkte entstehen. Zwei benachbarte Sattelpunkte würden jedoch, um die Stetigkeit zu wahren, eine der Hauptachsen der Sattelfläche zu einer Geraden machen. Mit einer endlichen Zahl Massen lässt sich aber kein solches Potential konstruieren.
Huggy



Anmeldungsdatum: 16.08.2012
Beiträge: 785

Beitrag Huggy Verfasst am: 24. Jun 2018 19:33    Titel: Antworten mit Zitat

jh8979 hat Folgendes geschrieben:
Die Gleichgewichtspunkte sind instabil, weil es immer Sattelpunkte sind (folgt aus dem Maximumprinzip für harmonische Funktionen).

Bei den Schlussfolgerungen daraus ist etwas Vorsicht geboten. Das Maximumprinzip gilt unabhängig von der Dimension im . Die Aussage, dass die Gleichgewichtspunkte Sattelpunkte sind, gilt aber nur im . Im können auch lokale Maxima auftreten. Das ist kein Widerspruch zum Maximumprinzip, denn ist zwar im eine harmonische Funktion, nicht aber im .

Wenn man 3 gleiche Massen auf die Ecken eines gleichseitigen Dreiecks setzt und das Potential nur in der Ebene der Massen betrachtet, dann hat es im Gleichgewichtspunkt ein lokales Maximum. Das ist auch instabil, weil es ein Maximum ist und kein Minimum. Zum Sattelpunkt wird der Gleichgewichtspunkt erst, wenn man die Konfiguration als Teil des betrachtet.
TomS
Moderator


Anmeldungsdatum: 20.03.2009
Beiträge: 18051

Beitrag TomS Verfasst am: 24. Jun 2018 22:50    Titel: Antworten mit Zitat

Könntest du das näher erläutern und durch Formeln darstellen?
_________________
Niels Bohr brainwashed a whole generation of theorists into thinking that the job (interpreting quantum theory) was done 50 years ago.
TomS
Moderator


Anmeldungsdatum: 20.03.2009
Beiträge: 18051

Beitrag TomS Verfasst am: 24. Jun 2018 22:52    Titel: Antworten mit Zitat

gnt hat Folgendes geschrieben:
TomS hat Folgendes geschrieben:
Anschaulich, aber kein Beweis

Vielleicht so: Wenn aus einem Sattelpunkt mehrere werden könnten, müssten, wenn man eine infinitesimale Masse hinzu fügt, mindestens zwei neue, infinitesimal benachbarte Sattelpunkte entstehen. Zwei benachbarte Sattelpunkte würden jedoch, um die Stetigkeit zu wahren, eine der Hauptachsen der Sattelfläche zu einer Geraden machen. Mit einer endlichen Zahl Massen lässt sich aber kein solches Potential konstruieren.

Ich verstehe nicht, was du meinst.

Fakt ist, dass beim Hinzufügen infinitesimaler Massen die “Entartung” von Gleichgewichtspunkten tatsächlich aufgehoben werden kann.

Das ist - so wie ich das sehe - gerade Gegenstand der von mir o.g. Bifurkationstheorie; und ich denke auch nicht, dass es einfacher geht.

_________________
Niels Bohr brainwashed a whole generation of theorists into thinking that the job (interpreting quantum theory) was done 50 years ago.
TomS
Moderator


Anmeldungsdatum: 20.03.2009
Beiträge: 18051

Beitrag TomS Verfasst am: 25. Jun 2018 07:31    Titel: Antworten mit Zitat

Hier ein möglicher Startpunkt für n Körper in Ringkonfiguration sowie ein weiterer Körper; außerdem einige Angaben zur Literatur

https://www.researchgate.net/profile/Antonio_Elipe/publication/222518621_Bifurcations_and_equilibria_in_the_extended_N-body_ring_problem/links/5a27cec2a6fdcc8e866e9201/Bifurcations-and-equilibria-in-the-extended-N-body-ring-problem.pdf?origin=publication_detail


Hier ein möglicher Startpunkt für das 3-Körper-Problem; Achtung - dabei wird die physikalisch sinnvolle Einschränkung vorgenommen, dass die drei Körper das dynamische Problem lösen, nicht dass sie statisch an einem Ort sitzen, was physikalisch eigtl. nicht möglich ist; außerdem einige Angaben zur Literatur

https://arxiv.org/abs/1012.4609


Generell: googeln zu n-body bifurcation; die o.g. Einschränkung der physikalisch erlaubten Bahnen gilt leider weiterhin, d.h. das hier vorliegende, wesentlich einfachere Problem wird nicht behandelt; n ist entweder gleich 4, wobei ein Körper dann sehr leicht sein muss, oder man findet explizit „3+1“ mit einem Körper infinitesimaler Masse.

M.E. ist eine Klärung mit dem Lehrer notwendig, um zu überlegen, ob das eine interessante Richtung sein könnte, und ob er unterstützen kann.

_________________
Niels Bohr brainwashed a whole generation of theorists into thinking that the job (interpreting quantum theory) was done 50 years ago.
Huggy



Anmeldungsdatum: 16.08.2012
Beiträge: 785

Beitrag Huggy Verfasst am: 25. Jun 2018 08:57    Titel: Antworten mit Zitat

TomS hat Folgendes geschrieben:
Könntest du das näher erläutern und durch Formeln darstellen?

Im hat man . Man erhält



Also greift das Maximumprinzip für diese Art von Potential im nicht. Für 3 gleiche Massen an den Ecken eines gleichseitigen Dreiecks habe ich mir das Potential in der Umgebung des Gleichgewichtspunkts plotten lassen. Es zeigte sich ein lokales Maximum.
TomS
Moderator


Anmeldungsdatum: 20.03.2009
Beiträge: 18051

Beitrag TomS Verfasst am: 25. Jun 2018 13:48    Titel: Antworten mit Zitat

OK, verstanden, das ist nochmal eine andere Betrachtungsweise.

Für die ursprüngliche Aufgabe ist das jedoch egal, denn gefragt ist nach Gleichgewichtspunkten = verschwindender erster Ableitung: von der zweiten Ableitung ist (noch) nicht die Rede.

Leider ist die praktische Relevanz der Aufgabe deswegen nicht gegeben, weil eine statische Massenkonfiguration betrachtet wird, nicht ein reale Lösung des 3-Körper-Problems mit der Diskussion von Gleichgewichtspunkten für eine vierte, infinitesimale Masse.

_________________
Niels Bohr brainwashed a whole generation of theorists into thinking that the job (interpreting quantum theory) was done 50 years ago.
TomS
Moderator


Anmeldungsdatum: 20.03.2009
Beiträge: 18051

Beitrag TomS Verfasst am: 25. Jun 2018 13:50    Titel: Antworten mit Zitat

Generell muss der TE klären, welchen Fragestellungen er sich widmen kann oder soll.
_________________
Niels Bohr brainwashed a whole generation of theorists into thinking that the job (interpreting quantum theory) was done 50 years ago.
jh8979
Moderator


Anmeldungsdatum: 10.07.2012
Beiträge: 8581

Beitrag jh8979 Verfasst am: 25. Jun 2018 16:30    Titel: Antworten mit Zitat

Huggy hat Folgendes geschrieben:
TomS hat Folgendes geschrieben:
Könntest du das näher erläutern und durch Formeln darstellen?

Im hat man . Man erhält



Also greift das Maximumprinzip für diese Art von Potential im nicht.

Richtig, aber weil die 1/r-Potentiale in zwei Dimensionen die Laplace-Gleichung nicht lösen, beschreiben sie auch nicht Gravitation oder Elektrostatik in zwei Dimensionen. Ich versteh also die Relevanz Deines Einwandes nicht.
Huggy



Anmeldungsdatum: 16.08.2012
Beiträge: 785

Beitrag Huggy Verfasst am: 25. Jun 2018 17:39    Titel: Antworten mit Zitat

jh8979 hat Folgendes geschrieben:

Richtig, aber weil die 1/r-Potentiale in zwei Dimensionen die Laplace-Gleichung nicht lösen, beschreiben sie auch nicht Gravitation oder Elektrostatik in zwei Dimensionen. Ich versteh also die Relevanz Deines Einwandes nicht.

Wenn sich mehrere Massen oder Ladungen einer in einer Ebene befinden, beschreiben die 1/r-Potentiale doch die Gravitation und Elektrostatik in dieser Ebene.
TomS
Moderator


Anmeldungsdatum: 20.03.2009
Beiträge: 18051

Beitrag TomS Verfasst am: 25. Jun 2018 17:47    Titel: Antworten mit Zitat

Es ist ein Unterschied, ob ich mich von vorneherein auf 2-dim. Probleme beschränke und konsequenterweise ein Potential der Form ln|r| betrachte, oder ob ich für ein 3-dim. Problem das 1/r Potential ansetze und zuletzt in die Ebene projiziere; wenn letzteres, dann muss ich m.E. auch bei der Taylorentwicklung und der Betrachtung der Stabilität berücksichtigen, dass ich eigtl. von drei Dimensionen herkomme und die entsprechende Klassifizierung der Hesse-Matrix und der Stabilität anzuwenden habe; andernfalls vermische ich Äpfel mit Birnen.

Ich halte das im vorliegenden Kontext jedoch für (fast) irrelevant, solange der TE nicht auf diese Diskussion einsteigt :-)

_________________
Niels Bohr brainwashed a whole generation of theorists into thinking that the job (interpreting quantum theory) was done 50 years ago.


Zuletzt bearbeitet von TomS am 25. Jun 2018 21:14, insgesamt 2-mal bearbeitet
jh8979
Moderator


Anmeldungsdatum: 10.07.2012
Beiträge: 8581

Beitrag jh8979 Verfasst am: 25. Jun 2018 19:41    Titel: Antworten mit Zitat

Huggy hat Folgendes geschrieben:
jh8979 hat Folgendes geschrieben:

Richtig, aber weil die 1/r-Potentiale in zwei Dimensionen die Laplace-Gleichung nicht lösen, beschreiben sie auch nicht Gravitation oder Elektrostatik in zwei Dimensionen. Ich versteh also die Relevanz Deines Einwandes nicht.

Wenn sich mehrere Massen oder Ladungen einer in einer Ebene befinden, beschreiben die 1/r-Potentiale doch die Gravitation und Elektrostatik in dieser Ebene.

Toms Antwort in meinen Worten:
Ja, die 1/r Potentiale beschreiben die Physik, in der von Dir betrachteten Eben, aber eben kein zweidimensionales System, da 1/r dort die Laplace-Gleichung nicht löst. Du willst also dreidimensionale Physik auf zwei Dimensionen beschränken, dafür bräuchtest Du irgendwelche Zwangskräfte (und dann ist das Problem fast trivial, da ich mit Zwangskräften fast beliebige Gleichgewichtspunkte konstruieren kann). Denn das von dir gefundene "Maximum/Minimum" ist in die dritte Richtung eben nicht stabil, sprich ein Sattelpunkt in drei Dimensionen.

Wenn Du ein wahres zweidimensionales System betrachtest, dann musst du auch die Laplace-Gleichung in zwei Dimensionen lösen mit allen Konsequenzen, die dann für harmonische Funktionen folgen.
Huggy



Anmeldungsdatum: 16.08.2012
Beiträge: 785

Beitrag Huggy Verfasst am: 25. Jun 2018 21:36    Titel: Antworten mit Zitat

jh8979 hat Folgendes geschrieben:
Du willst also dreidimensionale Physik auf zwei Dimensionen beschränken,

Will ich doch gar nicht.

Zitat:
Denn das von dir gefundene "Maximum/Minimum" ist in die dritte Richtung eben nicht stabil, sprich ein Sattelpunkt in drei Dimensionen.

Genau das sagte ich doch oben:

Huggy hat Folgendes geschrieben:

Zum Sattelpunkt wird der Gleichgewichtspunkt erst, wenn man die Konfiguration als Teil des betrachtet.

Offenbar ist mein obiger Hinweis, dass aus dem Maximumprinzip nicht folgt, dass das Potential einer ebenen Massenverteilung, wenn man es nur in der Ebene der Massen betrachtet, nicht schon als Funktion der 2 Koordinaten der Ebene einen Sattelpunkt haben muss, sondern erst bei Hinzunahme der dritten Koordinate, für niemanden verständlich.
jh8979
Moderator


Anmeldungsdatum: 10.07.2012
Beiträge: 8581

Beitrag jh8979 Verfasst am: 25. Jun 2018 22:51    Titel: Antworten mit Zitat

Dann sind wir uns in der Mathematik und Physik ja schonmal einig.
Huggy hat Folgendes geschrieben:

Offenbar ist mein obiger Hinweis, dass aus dem Maximumprinzip nicht folgt, dass das Potential einer ebenen Massenverteilung, wenn man es nur in der Ebene der Massen betrachtet, nicht schon als Funktion der 2 Koordinaten der Ebene einen Sattelpunkt haben muss,

Das würde ich auch nie behaupten.
Zitat:
sondern erst bei Hinzunahme der dritten Koordinate, für niemanden verständlich.

Jetzt wo Du das so schreibst ist es mit verständlich. Halte das aber für sinnlos, weil in diesem Kontext irrelevant... oder verpass ich irgendwas in Deiner Aussage? Bezieht sich das auf irgendwas anderes was hier (nicht von mir?) gesagt wurde?
TomS
Moderator


Anmeldungsdatum: 20.03.2009
Beiträge: 18051

Beitrag TomS Verfasst am: 26. Jun 2018 01:13    Titel: Antworten mit Zitat

Mich würde nach wie vor interessieren, ob für das vorliegende Problem allgemeine Beweise zur Anzahl der kritischen Punkte bekannt sind.
_________________
Niels Bohr brainwashed a whole generation of theorists into thinking that the job (interpreting quantum theory) was done 50 years ago.
Huggy



Anmeldungsdatum: 16.08.2012
Beiträge: 785

Beitrag Huggy Verfasst am: 26. Jun 2018 08:21    Titel: Antworten mit Zitat

jh8979 hat Folgendes geschrieben:

Jetzt wo Du das so schreibst ist es mit verständlich. Halte das aber für sinnlos, weil in diesem Kontext irrelevant... oder verpass ich irgendwas in Deiner Aussage? Bezieht sich das auf irgendwas anderes was hier (nicht von mir?) gesagt wurde?

Du verpasst nichts. Der Anlass für meinen Anmerkung war simpel. Bei 3 Massen auf den Ecken eines gleichseitigen Dreiecks hatte ein Maximum im Gleichgewichtspunkt. Nach deinem Hinweis auf das Maximumprinzip stutzte ich. Wie kann das sein? Da muss doch ein Sattelpunkt sein. Es dauerte etwas, bis ich darauf kam, dass ja ein Sattelpunkt ist und somit alles in bester Ordnung ist. Nun kam mir die hirnrissige Idee, dass auch andere vielleicht so begriffsstutzig sein könnten wie ich und so schrieb ich meine Anmerkung. Daher magst du also Recht haben, das sie sinnlos ist.
jh8979
Moderator


Anmeldungsdatum: 10.07.2012
Beiträge: 8581

Beitrag jh8979 Verfasst am: 26. Jun 2018 17:27    Titel: Antworten mit Zitat

Huggy hat Folgendes geschrieben:
jh8979 hat Folgendes geschrieben:

Jetzt wo Du das so schreibst ist es mit verständlich. Halte das aber für sinnlos, weil in diesem Kontext irrelevant... oder verpass ich irgendwas in Deiner Aussage? Bezieht sich das auf irgendwas anderes was hier (nicht von mir?) gesagt wurde?

Du verpasst nichts. Der Anlass für meinen Anmerkung war simpel. Bei 3 Massen auf den Ecken eines gleichseitigen Dreiecks hatte ein Maximum im Gleichgewichtspunkt. Nach deinem Hinweis auf das Maximumprinzip stutzte ich. Wie kann das sein? Da muss doch ein Sattelpunkt sein. Es dauerte etwas, bis ich darauf kam, dass ja ein Sattelpunkt ist und somit alles in bester Ordnung ist. Nun kam mir die hirnrissige Idee, dass auch andere vielleicht so begriffsstutzig sein könnten wie ich und so schrieb ich meine Anmerkung. Daher magst du also Recht haben, das sie sinnlos ist.

Ah, ok, jetzt versteh ich es. Thumbs up!
Neue Frage »
Antworten »
    Foren-Übersicht -> Mechanik